IMO Shortlist 2017 problem A1


Kvaliteta:
  Avg: 0,0
Težina:
  Avg: 6,0
Dodao/la: arhiva
3. listopada 2019.
LaTeX PDF

Let a_1,a_2,\ldots a_n,k, and M be positive integers such that \frac{1}{a_1}+\frac{1}{a_2}+\cdots+\frac{1}{a_n}=k\quad\text{and}\quad a_1a_2\cdots a_n=M.If M>1, prove that the polynomial P(x)=M(x+1)^k-(x+a_1)(x+a_2)\cdots (x+a_n)has no positive roots.

Izvor: https://www.imo-official.org/problems/IMO2017SL.pdf